Jump to content

the man's Content

There have been 595 items by the man (Search limited from 07-06-2020)



Sort by                Order  

#557922 Topic về Bất đẳng thức, cực trị THCS

Posted by the man on 05-05-2015 - 12:19 in Bất đẳng thức và cực trị

Các bạn giúp mình bài này nhé

Cho $x$, $y$ $>$$0$ thỏa mãn $x$+ $xy$+ $y$ $=$ $8$

Tìm GTNN của BT: P=$x^{3}$+$y^{3}$+ $x^{2}$+ $y^{2}$+ $5(x+y)$+ $\frac{1}{x}$+ $\frac{1}{y}$

$9=x+y+xy+1=(x+1)(y+1)\leq \left ( \frac{x+y+2}{2} \right )^2\Rightarrow (x+y+2)^2\geq 36\Rightarrow x+y\geq 4$

$P+40=(x^3+8+8)+(y^3+8+8)+(x^2+4)+(y^2+4)+\frac{19}{4}(x+y)+\frac{1}{4}(x+y)+\frac{4}{x+y}\geq 12x+12y+4x+4y+\frac{19}{4}(x+y)+2\geq 12.4+4.4+\frac{19}{4}.4+2=85\Rightarrow P\geq 45$

$minP=45 \Leftrightarrow x=y=2$




#557846 Topic về Bất đẳng thức, cực trị THCS

Posted by the man on 04-05-2015 - 19:38 in Bất đẳng thức và cực trị

Giúp mình với 
Với a,b,c >0, chứng minh rằng
$\frac{1}{a^{3}} + \frac{1}{b^{3}} + \frac{1}{c^{3}} \geq 9 (\frac{1}{a+2b}+\frac{1}{b+2c}+\frac{1}{c+2a})-6$

$\sum \frac{1}{a^3}+6=\sum \left ( \frac{1}{a^3}+1+1 \right )\geq \sum \frac{3}{a}=\sum \left ( \frac{1}{a}+\frac{1}{b}+\frac{1}{b} \right )\geq \sum \frac{9}{a+2b}\Rightarrow đpcm$




#557840 Topic về Bất đẳng thức, cực trị THCS

Posted by the man on 04-05-2015 - 19:31 in Bất đẳng thức và cực trị

Cho $x,y,z>0$ thỏa mãn : $x+y+z=2015$.

Chứng minh rằng:

$\sum \frac{2015x-x^2}{yz}+6\geq 2\sqrt{2}.\sum \sqrt{\frac{2015-x}{x}}$

$VT=\sum \frac{(x+y+z)x-x^2}{yz}+6=\sum \left ( \frac{x}{y}+\frac{x}{z} \right )+6=t+6$  

    Với $t=\sum \left ( \frac{x}{y}+\frac{x}{z} \right )$

$VP=2\sqrt{2}.\sum \sqrt{\frac{y+z}{x}}\leq 2\sqrt{2}.\sqrt{3(\sum \frac{y+z}{x})}=2\sqrt{6t}$

Như vậy ta cần chứng minh

   $t+6\geq 2\sqrt{6t}$   (điều này luôn đúng với bất đẳng thức cô-si)




#554886 Topic tổng hợp các bài toán về phương trình nghiệm nguyên.

Posted by the man on 18-04-2015 - 19:42 in Số học

 

2.Tìm nghiệm nguyên không âm: $x^{2}= y^{2}+\sqrt{y+1}$

 

Đặt $\sqrt{y+1}=a(a\geq 1)\Rightarrow y=a^2-1$

Thế và phương trình ta có 

               $x^2=(a^2-1)^2+a>(a^2-1)^2$

Mặt khác $x^2=a^4-(2a+1)(a-1)\leq (a^2)^2$ (do $a\geq1$)

   $\Rightarrow (a^2)^2\geq x^2>(a^2-1)^2\Rightarrow x=a^2$

   $\Rightarrow a=1\Rightarrow y=0\Rightarrow x=1$




#554981 Bất đẳng thức chuẩn bị cho kì thi THPTQG 2015-2016

Posted by the man on 19-04-2015 - 08:21 in Bất đẳng thức và cực trị

Cho : $x+y+z\geq 3$

Tìm min P =$\sum \frac{x^2}{yz+\sqrt{8+x^3}}$

Sử dụng Cô-si  $\sqrt{8+x^3}=\sqrt{(x+2)(x^2-2x+4)}\leq \frac{x^2-x+6}{2}\Rightarrow \frac{x^2}{yz+\sqrt{8+x^3}}\geq \frac{2x^2}{2yz+x^2-x+6}$

$\Rightarrow \sum \frac{x^2}{yz+\sqrt{8+x^3}}\geq 2.\frac{(x+y+z)^2}{x^2+y^2+z^2+2xy+2yz+2xz-x-y-z+18}$

                             $=2.\frac{(x+y+z)^2}{(x+y+z)^2-(x+y+z)+18}=2.\frac{t^2}{t^2-t+18}$      Với $t=x+y+z, t \geq3$

Nhận thấy dấu = xảy ra khi $t=3$ nên khi đó $\frac{t^2}{t^2-t+18}=\frac{3}{8}$

Ta chứng minh $\frac{t^2}{t^2-t+18}\geq \frac{3}{8}$

BĐT này tương đương với $5t^2+3t-54\geq 0$   (luôn đúng do $t \geq 3$)

$\Rightarrow Min P=\frac{3}{4}\Leftrightarrow x=y=z=1$




#554830 Bất đẳng thức chuẩn bị cho kì thi THPTQG 2015-2016

Posted by the man on 18-04-2015 - 15:13 in Bất đẳng thức và cực trị

Bài 1. Cho x, y, z >0 thỏa mãn x+y+z=3. Chứng minh rằng:

$\frac{x^{3}}{y^{3}+8}+\frac{y^{3}}{z^{3}+8}+\frac{z^{3}}{x^{3}+8}\geq \frac{1}{3}\geq \frac{1}{9}+\frac{2}{27}.(xy+yz+zx)$

 

Sử dụng bất đẳng thức cô-si:

$\frac{x^3}{(y+2)(y^2-2y+4)}+\frac{y+2}{27}+\frac{y^2-2y+4}{27}\geq \frac{x}{3}$

$\frac{y^3}{(z+2)(z^2-2z+4)}+\frac{z+2}{27}+\frac{z^2-2z+4}{27}\geq \frac{y}{3}$

$\frac{z^3}{(x+2)(x^2-2x+4)}+\frac{x+2}{27}+\frac{x^2-2x+4}{27}\geq \frac{z}{3}$

Cộng từng vế các BĐT trên ta có 
$\sum \frac{x^3}{y^3+8}+ \frac{x+y+z+6}{27}+\frac{x^2+y^2+z^2-2x-2y-2z+12}{27}\geq \frac{x+y+z}{3}$
$\Rightarrow \sum \frac{x^3}{y^3+8}\geq \frac{4}{9}-\frac{x^2+y^2+z^2}{27}$
  $=\frac{4}{9}-\frac{(x+y+z)^2-2(xy+yz+xz)}{27}=\frac{1}{9}+\frac{2}{27}(xy+yz+xz)$



#559040 $\boxed{\text{Chuyên Đề}}$ Bất đẳng thức - Cực trị

Posted by the man on 13-05-2015 - 06:45 in Bất đẳng thức và cực trị

Cho a, b, c > 0 và $a^{2}+b^{2}+c^{2}=3$. Tìm GTNN của $P=\frac{ab^{2}+bc^{2}+ca^{2}}{\left ( ab+bc+ca \right )^{2}}$

Dễ thấy $a+b+c\leq \sqrt{3(a^2+b^2+c^2)}=3$

Theo Cauchy-Schwarz 

$(a+b+c)(ab^2+bc^2+ca^2)\geq (ab+bc+ca)^2$

 $\Rightarrow \frac{ab^2+bc^2+ca^2}{(ab+bc+ca)^2}\geq \frac{1}{a+b+c}\geq \frac{1}{3}$




#558017 $\boxed{\text{Chuyên Đề}}$ Bất đẳng thức - Cực trị

Posted by the man on 05-05-2015 - 22:37 in Bất đẳng thức và cực trị

Sai rồi bạn tử các phân thức có là bình phương đâu mà Cauchy Schwartz

Bạn ấy làm tắt chút thôi

Bạn nhân cả tử và mẫu của mỗi phân thức ở VT lần lượt với $a,b,c$ là có bình phương ngay




#557879 Topic: Các bài toán về tính chia hết

Posted by the man on 04-05-2015 - 21:59 in Số học

Tìm các số tự nhiên n sao cho $A=2005^{n}+n^{2005}+2005n$ chia hết cho 3

Với $n=0$ không thỏa mãn

$A\equiv 1^n+n^{2005}+1.n\equiv n^{2005}+n+1  (mod3)$

Nếu $n\vdots 3\Rightarrow loại$

Nếu $n\equiv 1(mod3)\Rightarrow A\equiv 1+1+1\equiv 0(mod3)$

Nếu $n\equiv -1\Rightarrow A\equiv (-1)^{2005}+(-1)+1\equiv -1(mod3)$

Vậy $n=3k+1 (k \in N)$




#552766 $\boxed {\textbf{TOPIC}}$ Ôn thi VIOL...

Posted by the man on 09-04-2015 - 21:31 in Cuộc thi VIOlympic (Cuộc thi do Bộ giáo dục và đào tạo tổ chức)

1.các bạn giải phương trình này giúp mình với, mình mới tính ra số nghiệm là 4 chứ chưa giải kĩ các nghiệm ấy được

$\frac{x^{8}+x^{4}-2x^{2}+6}{x^{4}+2x^{2}+3}=11x^{2}-34$

Bạn để ý rằng $x^8+x^4-2x^2+6=(x^4+2x^2+3)(x^4-2x^2+2)$




#558068 $\boxed{\text{Chuyên Đề}}$ Phương trình vô tỉ - Hệ phương...

Posted by the man on 06-05-2015 - 17:32 in Phương trình, hệ phương trình và bất phương trình

Các bài toán phương trình vô tỉ trong các đề thi HSG tỉnh

 

 

           c) $\sqrt[3]{3x^{2}-x+2001}-\sqrt[3]{3x^{2}-7x+2002}-\sqrt[3]{6x-2003}=\sqrt[3]{2002}$

 

 

Câu c sử dụng nhân liên hợp

Ngoài ra còn có thể làm thế này 

Đặt $\left\{\begin{matrix}\sqrt[3]{3x^2-x+2001}=a & & & \\ \sqrt[3]{3x^2-7x+2002}=b & & & \\ \sqrt[3]{6x-2003}=c & & &\end{matrix}\right.\Rightarrow 2002=a^3-b^3-c^3$

Pt $\Leftrightarrow a-b-c=\sqrt[3]{a^3-b^3-c^3}$

Lập phương 2 vế rồi phân tích thành nhân tử ta được $(a-b)(a-c)(b+c)=0$

Đến đây xét 3 TH là ra 




#557207 $\boxed{\text{Chuyên Đề}}$ Phương trình vô tỉ - Hệ phương...

Posted by the man on 01-05-2015 - 07:28 in Phương trình, hệ phương trình và bất phương trình

Giải phương trình $\sqrt{5x-1}+\sqrt[3]{9-x}=2x^{2}+3x-1$

ĐK $x\geq \frac{1}{5}$

PT $\Leftrightarrow(\sqrt{5x-1}-2)+(\sqrt[3]{9-x}-2)=2x^2+3x-5 $

     $\Leftrightarrow \frac{5(x-1)}{\sqrt{5x-1}+2}+\frac{1-x}{\sqrt[3]{(9-x)^2}+2\sqrt[3]{9-x}+4}=(x-1)(2x+5)$

     $\Leftrightarrow (x-1).\left [ 2x+5+\frac{1}{\sqrt[3]{(9-x)^2}+2\sqrt[3]{9-x}+4}-\frac{5}{\sqrt{5x-1}+2} \right ]=0$

Do $\left\{\begin{matrix}2x+5\geq 2.\frac{1}{5}+5>\frac{5}{2}\geq \frac{5}{\sqrt{5x-1}+2} & & \\ \sqrt[3]{(9-x)^2}+2\sqrt[3]{9-x}+4\geq 0 & & \end{matrix}\right.\Rightarrow$ Biểu thức trong ngoặc vuông vô nghiệm

Như vậy $x-1=0 \Rightarrow x=1$




#555063 $\boxed{\text{Chuyên đề}}$: Phương tr...

Posted by the man on 19-04-2015 - 13:58 in Phương trình, hệ phương trình và bất phương trình

Để phương trình có 2 nghiệm phân biệt thì $\Delta =m^2-4m> 0\Leftrightarrow \begin{bmatrix} m<0\\ m>4 \end{bmatrix}$  $(1)$

Đặt $f(x)=x^2-mx+m=(x-x_1)(x-x_2)\Rightarrow f(-2)=(-2-x_1)(-2-x_2)\leq 0$

$\Leftrightarrow 4+3m\leq 0$

$\Leftrightarrow m\leq \frac{-4}{3}$     $(2)$

Kết hợp $(1)$ và $(2)$ thì $m\leq \frac{-4}{3}$

Sau khi tìm đk để pt có 2 nghiệm phân biệt thì cũng có thể làm thế này 

Pt có  2 nghiệm là $\frac{m-\sqrt{m^2-4m}}{2};\frac{m+\sqrt{m^2-4m}}{2}$

Từ đề bài ta có $\frac{m-\sqrt{m^2-4m}}{2}\leq -2<\frac{m+\sqrt{m^2-4m}}{2}$

Đến đây ta giải $m$




#555062 $\boxed{\text{Chuyên đề}}$: Phương tr...

Posted by the man on 19-04-2015 - 13:50 in Phương trình, hệ phương trình và bất phương trình

Bài 41: Tìm gt của a để 2 pt sau có ít nhất 1 nghiệm chung :

$x^{2}+ax+8=0(1)$

$x^{2}+x+a=0(2)$

 

Thế này sẽ đầy đủ hơn:

Gọi $x_1$ là nghiệm chung thế thì 

$\left\{\begin{matrix}x_1^2+ax_1+8=0& & \\ x_1^2+x_1+a=0 & &\end{matrix}\right.$

Với $a=1$ thì ta thấy không thỏa mãn đề bài 

Với $a\neq1$ trừ 2 vế của 2 PT ta được$x_1=\frac{a-8}{a-1}$

Thay vào PT thứ 2 thì $\left ( \frac{a-8}{a-1} \right )^2+\left ( \frac{a-8}{a-1} \right )+a=0\Leftrightarrow (a-8)^2+(a-8)(a-1)+a(a-1)^2=0\Leftrightarrow a^3-24a+72=0\Leftrightarrow a=-6$

Thử lại vào 2 pt thấy thỏa mãn

Vậy $a=-6$




#555065 $\boxed{\text{Chuyên đề}}$: Phương tr...

Posted by the man on 19-04-2015 - 14:17 in Phương trình, hệ phương trình và bất phương trình

Bài 44Tìm điều kiện của $m$ để phương trình sau có 4 nghiệm phân biệt

                                  $\left | x^2-2x+m \right |=x-1$

Bài 45.Cho phương trình  $x^3-(2m-1)x^2+(m^2-3m-2)x+2m^2+2m=0$

  a.Tìm $m$ để phương trình có đúng 2 nghiệm

  b.Xác định m để phương trình có 3 nghiệm phân biệt $x_1,x_2,x_3$ sao cho $S=x_1^2+x_2^2+x_3^2$ đạt GTNN

Bài 46.Cho $a \neq0, 2a+3b+6c=0$. tìm khoảng cách nhỏ nhất của 2 nghiệm của phương trình 

                                  $ax^2+bx+c=0$

Bài 47.Cho 2 số $a,b \neq0$ , khác nhau thỏa mãn $\frac{1}{a}+\frac{1}{b}=\frac{1}{2}$

       Chứng minh phương trình sau luôn có nghiệm $(x^2+ax+b)(x^2+bx+a)=0$

P/s:Thực ra bài 2,4 đều đưa về phương trình bậc 2 thôi  :icon6:  >:)




#555099 $\boxed{\text{Chuyên đề}}$: Phương tr...

Posted by the man on 19-04-2015 - 18:59 in Phương trình, hệ phương trình và bất phương trình

Điều kiện: $x\geq 1$

$(1)\Leftrightarrow \begin{bmatrix} x^{2}-3x+m+1=0;(2) \\ x^{2}-x+m-1=0;(3) \end{bmatrix}$

Phương trình $(1)$ có 4 nghiệm $\Leftrightarrow$ Phương trình $(2)$ và $(3)$ có 2 nghiệm phân biệt $\left\{\begin{matrix} \Delta_{2}=5-4m> 0 \\ \Delta_{3}=5-4m> 0 \end{matrix}\right.\Leftrightarrow m< \frac{5}{4}$

$(2)\Leftrightarrow \begin{bmatrix} x=\frac{3+\sqrt{5-4m}}{2} \\ x=\frac{3-\sqrt{5-4m}}{2} \end{bmatrix} ; (3)\Leftrightarrow \begin{bmatrix} x=\frac{1+\sqrt{5-4m}}{2} \\ x=\frac{1-\sqrt{5-4m}}{2} \end{bmatrix}$

Phương trình $(1)$ có 4 nghiệm phân biệt $\Leftrightarrow$ Phương trình $(1)$ có 4 nghiệm và 2 nghiệm của phương trình $(2)$ khác với 2 nghiệm của phương trình $(3)$

$\Leftrightarrow \left\{\begin{matrix} m< \frac{5}{4} \\ \frac{3+\sqrt{5-4m}}{2}\neq \frac{1+\sqrt{5-4m}}{2} \\ \frac{3+\sqrt{5-4m}}{2}\neq \frac{1-\sqrt{5-4m}}{2} \\ \frac{3-\sqrt{5-4m}}{2}\neq \frac{1+\sqrt{5-4m}}{2} \\ \frac{3-\sqrt{5-4m}}{2}\neq \frac{1-\sqrt{5-4m}}{2} \end{matrix}\right.\Leftrightarrow \left\{\begin{matrix} m<\frac{5}{4} \\ m\neq 1 \end{matrix}\right.$

Vậy phương trình có 4 nghiệm phân biệt khi $m\in (-\infty;\frac{5}{4})\setminus \left \{ 1 \right \}$

Đã có thiếu sót ở đây




#555105 $\boxed{\text{Chuyên đề}}$: Phương tr...

Posted by the man on 19-04-2015 - 19:28 in Phương trình, hệ phương trình và bất phương trình

Hình như thiếu sót là cái điều kiện $x\geq 1$ ban đầu đúng không ? Thực sự thì mình có để ý qua nhưng khi giải như vyậ... ta được m thuộc rỗng có vẻ hư cấu quá @@

Đúng rồi. Không tồn tại giá trị của m thỏa mãn đề bài




#554512 Cauchy-Schwarz

Posted by the man on 16-04-2015 - 22:48 in Bất đẳng thức và cực trị

Cho a, b, c > 0. Chứng minh rằng $\frac{1}{a(b+1)}+\frac{1}{b(c+1)}+\frac{1}{c(a+1)}\geq \frac{3}{abc+1}$

Bđt cần chứng minh tương đương với $\frac{1+abc}{a(1+b)}+\frac{1+abc}{b(c+1)}+\frac{1+abc}{c(a+1)}\geq 3$

$\Leftrightarrow \frac{1+abc+a(1+b)}{a(b+1)}+\frac{1+abc+b(c+1)}{b(c+1)}+\frac{1+abc+c(1+a)}{c(a+1)}\geq 6$

$\Leftrightarrow [\frac{a+1+ab(c+1)}{a(b+1)}+\frac{b+1+bc(a+1)}{b(c+1)}+\frac{c+1+ca(b+1)}{c(a+1)}\geq 6$

$\Leftrightarrow [\frac{a+1}{a(b+1)}+\frac{a(b+1)}{a+1}]+[\frac{b+1}{b(c+1)}+\frac{b(c+1)}{b+1}]+[\frac{c+1}{c(a+1)}+\frac{c(a+1)}{c+1}]\geq 6$ (*)

Theo cô-si $\frac{a+1}{a(b+1)}+\frac{a(b+1)}{a+1}\geq 2$

    tương tự$\frac{b+1}{b(c+1)}+\frac{b(c+1)}{b+1}\geq 2$

                     $\frac{c+1}{c(a+1)}+\frac{c(a+1)}{c+1}\geq 2$

  $\Rightarrow (*)$ luôn đúng

 Dấu = xảy ra khi a=b=c=1




#554825 Cauchy-Schwarz

Posted by the man on 18-04-2015 - 14:54 in Bất đẳng thức và cực trị

Cho x,y $\epsilon$ R và 21x^2-36xy+44y^2 $\leqslant$ 27. CMR: x+2y $\geq$ -3

Đặt $x+2y=t \Rightarrow x=t-2y$. Thay vào đầu bài ta có 

 $21(t-2y)^2-36y(t-2y)+44y^2 -27 \leq 0$

Thu gọn và nhóm hợp lí ta được $2(10y-3t)^2+3t^2-27 \leq0$

           $ \Rightarrow 3t^2-27 \leq0 \Rightarrow -3 \leq t \leq3$




#556424 Cauchy-Schwarz

Posted by the man on 26-04-2015 - 15:08 in Bất đẳng thức và cực trị

Cho $x+y+z\leq \frac{3}{2}$ và x, y, z > 0. Tìm GTNN của $P=\left ( \frac{1}{x}+\frac{1}{y}+3 \right )\left ( \frac{1}{y}+\frac{1}{z}+3 \right )\left ( \frac{1}{z}+\frac{1}{x}+3 \right )$

Đặt $\frac{1}{x}+\frac{1}{y}=a,\frac{1}{y}+\frac{1}{z}=b,\frac{1}{x}+\frac{1}{z}=c$

  $\Rightarrow P=(a+3)(b+3)(c+3)=abc+9(a+b+c)+3(ab+bc+ca)+27$

  $\geq abc+27+27\sqrt[3]{abc}+9\sqrt[3]{a^2b^2c^2}$

$xyz\leq \left ( \frac{x+y+z}{3} \right )^3=\frac{1}{8}\Rightarrow abc=\left ( \frac{1}{x}+\frac{1}{y} \right )\left( \frac{1}{y}+\frac{1}{z} \right )\left( \frac{1}{x}+\frac{1}{z} \right ) \geq \frac{8}{xyz}\geq 64$

$\Rightarrow P\geq 64+27+144+108=343$




#560931 Topic các bài về số nguyên tố

Posted by the man on 22-05-2015 - 16:35 in Số học

Tìm các số nguyên tố p, q và số nguyên x thỏa mãn $x^{5}+px+3q=0$

 Từ giả thiết ta có:

      $x(x^4+p)=-3q\Rightarrow 3q\vdots x\Rightarrow x\in \left \{ \pm 1;\pm 3;\pm q;\pm 3q \right \}$  

Dễ thấy $x<0\Rightarrow x\in \left \{ -1;-3;-q;-3q \right \}$

- TH1: $x=-1$. Thay vào đầu bài ta có:

                                                 $3q=p+1$

   +Nếu $p=2 \Rightarrow q=1$ (không thỏa mãn)

   +Nếu $q=2 \Rightarrow p=5$ (thỏa mãn)

   +Nếu $p,q>2 \Rightarrow p,q$  lẻ $\Rightarrow 3q$   lẻ, $p+1$   chẵn   (không thỏa mãn)

-TH2: $x=-3$. Thay vào đầu bài ta có:

                                          $q=p+81$    $\Rightarrow q>2$   nên  q  lẻ

   +Nếu $p=2 \Rightarrow q=83$   ( thỏa mãn)

   +Nếu $p>2 \Rightarrow p$  lẻ $\Rightarrow p+81$   chẵn    (không thỏa mãn)

-TH3: $x=-q$. Thay vào đầu bài ta có:

                                         $q^4+p=3$ ( vô lí vì $p,q \geq 2$)

-TH4: $x=-3q$. Thay vào giả thiết ta có:

                                         $81q^4+p=1$ (vô lí vì $p,q \geq 2$)

 

                                       Vậy $\boxed{\textrm{(x;p;q)=(-1;5;2);(-3;2;83)}}$

 

  




#569423 Topic tổng hợp một số bất đẳng thức trong kì thi MO các nước

Posted by the man on 02-07-2015 - 11:03 in Bất đẳng thức - Cực trị

Đặt $(a;b;c)=(\frac{x}{y};\frac{y}{z};\frac{z}{x})$ , đưa bđt cần cm thành

$\sum \frac{x^{2}}{x^{3}+x^{2}+1}\leq\sum \frac{x}{2x+1}$

Hay $\sum(x-1)^{2}(x+1)x\geq 0$  (đúng)

Vậy bất đẳng thức đc cm

Dấu đẳng thức xảy ra khi $a=b=c=1$ 

Thế thì $abc=1$ à




#569421 Topic tổng hợp một số bất đẳng thức trong kì thi MO các nước

Posted by the man on 02-07-2015 - 11:01 in Bất đẳng thức - Cực trị

Bài 153: (Việt Nam TST 2005) 

Cho $a,b,c>0$. Chứng minh: $\left ( \frac{a}{a+b} \right )^3+\left ( \frac{b}{b+c} \right )^3+\left ( \frac{c}{c+a} \right )^3 \geq \frac{3}{8}$

$P=\left ( \frac{a}{a+b} \right )^3+\left ( \frac{b}{b+c} \right )^3+\left ( \frac{c}{c+a} \right )^3=\frac{1}{\left ( 1+\frac{b}{a} \right )^3}+\frac{1}{\left ( 1+\frac{c}{b} \right )^3}+\frac{1}{\left ( 1+\frac{a}{c} \right )^3}$

    $=\frac{1}{(1+x)^3}+\frac{1}{(1+y)^3}+\frac{1}{(1+z)^3}$       Với  $\frac{b}{a}=x,\frac{c}{b}=y,\frac{a}{c}=z\rightarrow xyz=1$

Sử dụng bất đẳng thức Am-GM:

    $\frac{1}{(1+x)^3}+\frac{1}{(1+x)^3}+\frac{1}{8}\geq \frac{3}{2}.\frac{1}{(1+x)^2}\rightarrow \frac{1}{(1+x)^3}\geq \frac{3}{4}.\frac{1}{(1+x)^2}-\frac{1}{16}$

    $\rightarrow P\geq \frac{3}{4}.\left [ \frac{1}{(1+x)^2}+\frac{1}{(1+y)^2}+ \frac{1}{(1+z)^2}\right ]-\frac{3}{16}$

Ta cần chứng minh :  

            $\frac{1}{(1+x)^2}+\frac{1}{(1+y)^2}+ \frac{1}{(1+z)^2}\geq \frac{3}{4}$    (*)

Ta có bài toán quen thuộc:

Với  $m,n,p,q$  là các số dương có tích bằng 1 thì  

            $\frac{1}{(1+m)^2}+ \frac{1}{(1+n)^2}+\frac{1}{(1+p)^2}+ \frac{1}{(1+q)^2}\geq 1$

Áp dụng vào bài toán với  $m=x,n=y,p=z,q=1$ ta có  (*)

Vậy bài toán được chứng minh xong




#569349 Topic tổng hợp một số bất đẳng thức trong kì thi MO các nước

Posted by the man on 01-07-2015 - 19:59 in Bất đẳng thức - Cực trị

Bài 163(Spain TST): Cho a,b,c>0: $a^2+b^2+c^2=3$. CMR:

$\frac{a}{a^2+b^3+c^2}+\frac{b}{b^2+c^3+a^2}+\frac{c}{c^2+a^3+b^2}\leq 1$

Sử dụng bất đẳng thức Holder:

    $(a^2+b^3+c^2)(a^2+b^3+c^2)(a^2+1+c^2)\geq (a^2+b^2+c^2)^3=27$

    $\rightarrow a^2+b^3+c^2\geq \frac{3\sqrt{3}}{\sqrt{a^2+c^2+1}}\rightarrow \frac{a}{a^2+b^3+c^2}\leq \frac{a\sqrt{a^2+c^2+1}}{3\sqrt{3}}$

    $\rightarrow \sum \frac{a}{a^2+b^3+c^2}\leq \frac{\sum a\sqrt{a^2+c^2+1}}{3\sqrt{3}}$

Sử dụng bất đẳng thức  Cauchy-Schwarz:

    $\sum a\sqrt{a^2+c^2+1}\leq \sqrt{(a^2+b^2+c^2)(2a^2+2b^2+2c^2+3)}=3\sqrt{3}$

Từ đó ta có điều phải chứng minh




#569410 Topic tổng hợp một số bất đẳng thức trong kì thi MO các nước

Posted by the man on 02-07-2015 - 09:44 in Bất đẳng thức - Cực trị

Bài 165 (Mediterranean Mathematical Competition 2009).Chứng minh rằng với mọi  $a,b,c $ dương ta luôn có:

$$\sum \frac{ab}{a^2+ab+b^2}\leq \frac{a}{2a+b}+\frac{b}{2b+c}+\frac{c}{2c+a}$$